LSAT and Law School Admissions Forum

Get expert LSAT preparation and law school admissions advice from PowerScore Test Preparation.

 Administrator
PowerScore Staff
  • PowerScore Staff
  • Posts: 8927
  • Joined: Feb 02, 2011
|
#36650
Complete Question Explanation

Justify the Conclusion. The correct answer choice is (A)

The physician-author of this passage explains that the increase in blood pressure that commonly
comes with aging is caused by a calcium deficiency, which is caused by an inability to absorb
calcium, which, in turn, is frequently caused by a vitamin D deficiency.

The doctor then points out that the calcium deficit is equal to the amount of calcium found in a glass
of milk, and concludes that drinking milk can therefore lower some older people’s blood pressure.
The leap that it took to arrive at the conclusion was an assumption that these older people are able
to use that calcium (for example, there could be rocks that contain as much calcium as is lost with
aging, but that does not necessarily mean that eating such rocks would solve a vitamin D deficiency
in an older person).

The stimulus is followed by a Justify question, so the correct answer choice will justify the
physician’s conclusion that the calcium lost in some older people can be replaced by the calcium in
milk.

Answer choice (A): This is the correct answer choice. If the calcium in milk is to compensate
for the calcium deficiency that many older people suffer from, then those people must be able to
actually absorb the calcium. This choice provides that milk has sufficient quantities of vitamin D,
or whatever else is required, to allow older people to generally absorb the milk’s calcium, thus
replacing the deficit that commonly accompanies aging.

Answer choice (B): This choice does not justify the author’s conclusion, because even if milk is
guaranteed not to increase blood pressure, that is not relevant, as it does not establish that older
people will be able to use the calcium found in milk to compensate for the calcium deficiency that
many older people experience.

Answer choice (C): This choice provides that drinking milk does not contribute to a calcium
deficiency in older people. While this choice establishes that drinking milk won’t make the
deficiency worse than it already was, it does not justify the author’s conclusion that drinking milk
can compensate for the deficiency which already exists in many older people, thereby lowering
blood pressure.

Answer choice (D): In order to justify the conclusion from the stimulus, that drinking milk can
reduce blood pressure in many older people, the correct answer choice must confirm that the calcium
in milk can be absorbed by older people, to cure this deficiency and reduce their blood pressure.
Since the author’s conclusion is not justified by this choice, it cannot be the correct answer.

Answer choice (E): This choice provides that all people with a vitamin D deficiency also have a
calcium deficiency. Since this doesn’t confirm the author’s conclusion that milk can lower blood
pressure in older people, it cannot be the correct answer to this Justify question.
 Garrett K
  • Posts: 28
  • Joined: Jul 28, 2014
|
#16674
Hello,

I have two questions....

I understand why A is correct, but why is B not correct. If the main point of the argument is that milk will help lower BP, then why would B be incorrect considering that this answer choice rules out an increase in BP from another substance?

Sorry for asking so many questions on the forum today....

Thanks,
Garrett
 Emily Haney-Caron
PowerScore Staff
  • PowerScore Staff
  • Posts: 577
  • Joined: Jan 12, 2012
|
#16699
Hi Garrett,

B is not sufficient to justify the conclusion; it tells us milk won't make their blood pressure go up, but doesn't actually tell us whether it will be able to do anything to lower it. A is the only answer that fills the hole in the reasoning by explaining that the calcium in milk will be absorbed.

Hope that helps!
 lday4
  • Posts: 44
  • Joined: May 05, 2016
|
#24280
I'm having trouble understanding why A is a better choice over C. My thought was that A didn't need to be assumed because even if there wasn't enough vitamin D in the milk it doesn't mean that all older people have a vitamin D deficiency and needed the vitamin D in the milk. The stimulus says that calcium deficiency is frequently caused by, not always caused by VD deficiency.

C made sense to me because if an older person is drinking milk for their calcium deficiency, but milk contributes to a VD deficiency then no older person would be able to make up for their calcium deficiency by drinking milk - which destroys the conclusion.

Thanks!
 David Boyle
PowerScore Staff
  • PowerScore Staff
  • Posts: 836
  • Joined: Jun 07, 2013
|
#24291
lday4 wrote:I'm having trouble understanding why A is a better choice over C. My thought was that A didn't need to be assumed because even if there wasn't enough vitamin D in the milk it doesn't mean that all older people have a vitamin D deficiency and needed the vitamin D in the milk. The stimulus says that calcium deficiency is frequently caused by, not always caused by VD deficiency.

C made sense to me because if an older person is drinking milk for their calcium deficiency, but milk contributes to a VD deficiency then no older person would be able to make up for their calcium deficiency by drinking milk - which destroys the conclusion.

Thanks!

Hello,

C is tempting, but the stimulus doesn't say that older folks drink exactly one glass of milk. Also, even if drinking one glass helps create a vitamin D deficiency, maybe there's not enough of a tendency toward deficiency, to outweigh the amount of vitamin D in the milk. (And maybe other factors somehow make up for that tendency toward deficiency.)
But what is needed to justify the conclusion is that there is enough vitamin D in the milk. (Not to mention any other substances needed, which answer C doesn't address.) What you mention above, about "it doesn't mean that all older people have a vitamin D deficiency and needed the vitamin D in the milk", is not relevant here, since the stimulus didn't assert that they all had a deficiency.

David
 Lsat_student2019
  • Posts: 8
  • Joined: Aug 09, 2019
|
#67724
Hi, in reviewing this question I was just wondering if B is more fitting for an assumption question. I understand how it does not justify the conclusion, but do see how it could damage it if correct. Do you think this answer is more favorable in an assumption rather than justify question? I am trying to understand why I picked it and I think it was because of that! Thank you.
User avatar
 Dave Killoran
PowerScore Staff
  • PowerScore Staff
  • Posts: 5848
  • Joined: Mar 25, 2011
|
#67726
Lsat_student2019 wrote:Hi, in reviewing this question I was just wondering if B is more fitting for an assumption question. I understand how it does not justify the conclusion, but do see how it could damage it if correct. Do you think this answer is more favorable in an assumption rather than justify question? I am trying to understand why I picked it and I think it was because of that! Thank you.
Looking at (B) strictly as if this were an Assumption question, it would not qualify as an assumption of the argument. The author doesn't presume that milk contains no substances at all that would be likely to increase blood pressure, but instead presumes that milk doesn't contain so much of any such substance that it counteracts or significantly impairs the ability of milk as a whole to make up the deficiency and help blood pressure. In other words, this answer goes too far to be an assumption in saying "does not contain any substance." this is a close distinction, but the LSAT definitely expects students to see that difference (if this were an assumption question, that is :-D ).

Please let me know if that helps. Thanks!
User avatar
 cornflakes
  • Posts: 48
  • Joined: Feb 19, 2021
|
#85518
Hi Powerscore - I arrived at A here by noticing that B and C didn't necessarily justify the argument - C looked enticing for a while though and I'm wondering if its a necessary assumption of the argument (if that were the question type being asked).

For instance, if older people drinking one glass of milk per day would contribute to a deficiency in the active form of vitamin D needed in order for the body to absorb the calcium in that very milk, then this would weaken the idea that some people can lower their blood pressure by drinking milk because they would be less likely to have the ability to absorb the calcium in it that is needed to lower the blood pressure in the first place.

Let me know if this is good analysis or if it would not work for a necessary 2N question.
 Jeremy Press
PowerScore Staff
  • PowerScore Staff
  • Posts: 1000
  • Joined: Jun 12, 2017
|
#85979
Hi cornflakes,

Absolutely, and nice analysis! If you run the Assumption Negation Technique on answer choice C, the negated form of the answer destroys the conclusion. That's because if drinking that glass of milk does contribute to the deficiency, then you can't use the glass of milk to make up for the deficiency and lower your blood pressure. Incidentally, that's a really good example of why you shouldn't use the Assumption Negation Technique on Justify or Strengthen questions. It could lead to a wrong answer (a necessary, rather than a sufficient, assumption). Keep up the good work!

Get the most out of your LSAT Prep Plus subscription.

Analyze and track your performance with our Testing and Analytics Package.